PMP Exam Study

Ace your homework & exams now with Quizwiz!

You are meeting with the quality assurance (QA) manager to discuss the results of the quality audit of your project. The QA manager informs you that your team has been following quality procedures that are not particularly helpful, and as a result might be increasing the likelihood for defects. The QA manager recommends revisions to the quality procedures. You feel that the team is following the quality policies that were originally set forth, and they are doing a good job. How should you handle this situation? A. You should make a note in your lessons learned document B. Go to the functional managers of your team members and ask them to ensure the team members follow the new procedures suggested C. You should analyze the suggested procedures, then submit a change request D. Do nothing because your team is already doing a good job

C. You should analyze the suggested procedures, then submit a change request The correct course of action when you identify a possible change is to analyze the impact of the change and then present a change request to the change control board. Choice B might very well be the end result of this, but it's important to take your change request to the change control board for approval. Choice D is obviously not a good way to handle this (even though it might be your first instinct).

You are a project manager working on contract for Ralph's Toy shop. Your project involves implementing a new department in 25 locations across the country as a pilot to determine if this will be a profitable new service. You've identified two alternative methods of implementing the pilot. Alternative A's initial investment equals $596,000. The PV of the expected cash inflows is $299,000 in year 1 and $301,000 in year 2. The cost of capital is 14 percent. Alternative B's initial investment equals $625,000. The PV of Alternative B's expected cash inflows is $321,000 in year 1 and $301,000 in year 2. The cost of capital is 10 percent. A. Alternative A will earn a return of at least 14 percent. B. Alternative A will earn a return of at least 25 percent. C. The return is not known for either Alternative A or Alternative B. D. Both alternatives are equally viable choices.

A. Alternative A will earn a return of at least 14 percent. Since the Present Value of both alternatives is already given, you add up the PV of year 1 and year 2 in both alternatives, subtract our their respective initial investments, and then you have their Net PV's. Since NPV is positive for project A, that makes answer choice A a true statement. An NPV equal to zero would result in a 14% return, and thus, Project A must earn a return greater than 14%

You are a project manger on a multi-year transportation project for the state. You are about two-thirds complete with the project and, so far, the project has progressed well. You continue to work with your stakeholders throughout the project and they have been engaged and helpful in championing your project, identifying issues and providing recommendations. However, recently, you have noticed new stakeholders appear, which you were not expecting and have not identified. This has created some tension on the team; team members are becoming concerned a requirement may have been missed. What should you do? A. Review your stakeholder engagement plan to ensure you are identifying and engaging stakeholders correctly. B. Refer to project documents such as the change log, issue log and requirements documentation. C. Review the communications management plan to determine if you have any gaps in your communicating with your stakeholders. D. Review your project management plan as it is your overall guide for how to manage your project.

B. Refer to project documents such as the change log, issue log and requirements documentation. The problem is you have not identified all the stakeholders. It is unlikely any project documents will be an input for initial stakeholder identification. However, stakeholder identification occurs throughout the project. Once the project is initially planned, more documents become available and are used throughout the project. Project documents to be considered as inputs for Identifying Stakeholders include the change log, issue log and requirements documentation. Reviewing a plan will not necessarily directly address the issue of not identifying stakeholders throughout the project (Choices A, C and D).

You are a project manager for a wedding-planning company. Since every wedding is unique, your organization believes in managing each wedding as a project. You've come up with a great idea for a new event that you're certain customers will love and which will also generate profit for the company. Your boss asked you to investigate the best alternative methods for implementing the new idea and come back with a suggested solution. You discover that Alternative A could yield revenues of $100,000 in two years, while Alternative B could yield revenues of $107,000 in three years. The finance manager told you to use eight percent as the cost of capital. Which project should you choose and why? A. Alternative A, because the discounted cash flows are $85,734, while the discounted cash flows for Alternative B are $84,940. B. Alternative A, because the discounted cash flows are $85,940, while the discounted cash flows for Alternative B are $83,352. C. Alternative B, because the discounted cash flows are $85,700, while the discounted cash flows for Alternative A are $83,352. D. Alternative A, because the discounted cash flows are $96,457, while the discounted cash flows for Alternative B are $84,677.

A. Alternative A, because the discounted cash flows are $85,734, while the discounted cash flows for Alternative B are $84,940. This question requires discounted cash flow analysis to compare the value of Alternative A to Alternative B. The present value formula for Alternative A is calculated as follows: $100,000 / (1 + .08)^2 = $85,734. Alternative B is calculated this way: $107,000 / (1 + .08)^3 = $84,940. The present value formula is being used, as opposed to NPV, because the question didn't state revenue was being received each year. Instead it states the revenue in two and three years, so assuming at the end of year 2 for Alternate A and year 3 for Alternate B.

A project is being implemented. The research and development department communicates to the project manager that the product's requirements have changed. These changes were not anticipated, may significantly influence potential sales numbers, and could impact project scope. What is the appropriate action for the project manager to take? A. Anticipate risk responses and modify the risk register accordingly. B. Use forecasting methods. C. Plan to use the contingency reserve and update the risk mitigation plan. D. Utilize methods such as earned value management (EVM).

A. Anticipate risk responses and modify the risk register accordingly. These new requirements may pose additional risks. These risks need to be documented and appropriate responses need to be identified.

Which of the following is not included in configuration management? A. Automatic change request approvals B. Controlling changes to project deliverables C. Communicating changes to those performing the affected work D. Identification of the functional and physical attributes of the project deliverables

A. Automatic change request approvals Choices B, C, and D are all part of configuration management. Change requests are not and should not be automatically approved.

You are managing a large project team engaged in the development of a new hybrid car. It is a virtual team with facilities in the United States, India, China and Spain. The project team is divided into sub-groups, such as design, manufacturing, etc. As the project manager, your greatest challenge will likely be with which of the following: A. Coordinating the activities of the project team members and integrating the results of the various sub-units B. Controlling quality since you have multiple production facilities C. Controlling cost since the project team members are global D. Managing stakeholder engagement and effectively communication with stakeholders from different cultures

A. Coordinating the activities of the project team members and integrating the results of the various sub-units In this scenario, you have multiple, cross-functional resources you are managing and must coordinate and integrate their work and work results. There will be complexities with schedule dependencies, configuration management, and communication amongst the various groups, among others. Answer A best reflects these challenges.

You are the project manager for the construction of a mall. You are working through the Planning Process Group and reviewing cost components of your project. Before you begin the Determine Budget process, what is needed? A. Cost Estimates, Basis of Estimates and Project Schedule B. Estimate Activity Duration and Basis for Estimates C. Estimate at Completion and Budget at Completion D. Parametric model used to arrive at the costs submitted

A. Cost Estimates, Basis of Estimates and Project Schedule Cost Estimates, Basis of Estimates and Project Schedule are inputs to the Determine Budget process.

You are in the process of executing your project and need to know when it will be completed. You have the following activity durations on your project: A=8, B=14, C=6, D=6, E=12, F=6. You also know that D is dependent on A and B. Task F is a successor of D and E. Task C is a predecessor of task E. Based on this information what is the critical path and what is the float of task A? A. Critical path is BDF and the float of task A is 6 B. Critical path is ADF and the float of task A is 0 C. Critical path is CEF and the float of task A is 0 D. Critical path is BDF and the float of task A is 2

A. Critical path is BDF and the float of task A is 6 For this question you need to draw out the schedule network diagram based on the information provided. You would have the following paths: A to D, B to D, C to E, D to F and E to F. You would then need to use the durations provided to do a forward and backwards path analysis. The end result is that the critical path is BDF and the float of task A is 6.

Which of the following processes falls under Project Integration Management? A. Develop project charter B. Validate Scope C. Change control system D. Close Procurements

A. Develop project charter Develop Project charter is one of the processes in Integration Management. Note that there are Integration Management processes in each of the 5 process groups.

Your project team does not understand the fundamentals of project management. You feel this has the potential to harm your project. You decide to put together a half-day training session to teach project management basics. Which of the following have you achieved by having this training? A. Develop your project team B. Avoid Risk C. Acquire Resources D. Analyze Risk

A. Develop your project team This is an example of developing your project team because you are enhancing the team's overall competencies and skills. Note that B is incorrect because avoiding a risk means taking it off the table completely, but project failures could still occur notwithstanding the training.

A software vendor is behind on a key project deliverable. There is a 21% buffer in the budget for cost overruns. The project manager needs to mitigate the issue. Which action should the project manager take next? A. Evaluate alternative software vendors that can help project deliverables despite resource constraints. B. Remove the current vendor and have the new vendor send a contract. C. Ask the vendor to meet to discuss options for helping them get caught up. D. Accept the cost overrun and update the project schedule.

A. Evaluate alternative software vendors that can help project deliverables despite resource constraints. The first step is analysis and evaluation of alternate options. After that, specific and appropriate action can be taken. PMBOK Guide Sixth Edition (2017) PMI/PMI/12.3 Control Procurements/p. 492

Which of the following processes does not result in updates to the Risk Register? A. Identifying lessons learned in risk management during project closure B. Perform Qualitative Risk Analysis C. Plan Risk Responses D. Monitor and Control Risks

A. Identifying lessons learned in risk management during project closure Lessons identified during closure would not be captured in a Risk Register, which is used to document and track risks during project performance. The remaining answers all have risk register updates as outputs.

As a project manager you will have to possess many skills to aid in the successful delivery of your projects. Which of the following critical skills is often the hardest to master? A. Interpersonal Skills (Soft Skills) B. Communication Skills C. Organization Skills D. Technical Skills

A. Interpersonal Skills (Soft Skills) Soft skills, such as tailoring your management style to best accommodate the widely varying personalities of your team members, are often the most difficult to master. This is because these skills encompass project-specific leadership, influencing, and effective decision making (techniques that vary widely depending on culture and other project factors).

A project is 7 months into execution. The project manager determines that the cost performance index (CPI) is .80. According to a trend analysis, the CPI has a downward trend. What is the next step that the project manager should take? A. Issue a change request for cost re-baselining. B. Ask for additional resources. C. Crash the project schedule. D. Utilize the management reserve.

A. Issue a change request for cost re-baselining. Since the project is over budget and the trend analysis show that will continue, the best course of action is to re-baseline the cost of the project and manage to that.

In agile, what is the most important reason to build, test, and review small increments? A. It evaluates and delivers continuous benefits and value B. It reduces future testing of the work product at the time of release C. It promotes relationship building between the agile team and business people D. It reduces issues and risk

A. It evaluates and delivers continuous benefits and value Option A is correct. By building, testing, and reviewing small increments, business value is evaluated and delivered continuously. The first agile principle is about delivering business value early and continuously.

Your team has been together for a few years and is known as a highly efficient team throughout the organization. A team member who is new to the team lacks the tacit knowledge required to create some of the features. It is late in the work day and you notice the employee struggling with some work. How should you proceed? A. Offer your assistance with the creation of the feature B. Document the problem in the issue log and follow up the next business day C. Document it on the risk register and develop a potential response D. Tell the employee to follow the project management plan

A. Offer your assistance with the creation of the feature Servant leadership is the virtue of a project manager or scrum team. We should always support the team through mentoring, encouragement, support, and on-the-job training if required.

Your management decided to terminate the project due to persistent budget and schedule overruns by the supplier. What should you do immediately? A. Proceed with the closure B. Call a meeting with management to try to resurrect the project C. Discuss it with the team members and present a revised project management plan to management D. Finish completing the deliverables that are nearly complete, then perform close-out procedures

A. Proceed with the closure The project was terminated, so the next step is to formally close the project. This includes gathering and organizing relevant documentation, closing out procurements, and releasing the team.

You are the project manager for a large consumer goods company. You are currently managing a project to expand into the farm to fork area. You are working with local farmers and realize that you will need to shift your business model and include vendors to assist with immediate delivery. Inputs to Control Procurements include which of the following? A. Procurement Documents, Agreements, Work Performance Data, Approved change request B. Agreements, work results, scope request and seller invoices C. Agreements, work results, contract change control system D. Agreements, work breakdown structure, change request and seller invoices

A. Procurement Documents, Agreements, Work Performance Data, Approved change request Inputs to the Control Procurements process are procurement documents, project management plan, agreements, work performance data and approved changes request.

You have four possible projects but can only choose one. Project A is being done over a six-year period and has a net present value (NPV) of $70,000. Project B is being done over a three-year period and has an NPV of $30,000. Project C is being done over a ten-year period and has an NPV of $40,000. Project D is being done over a one-year period and has a NPV of negative $160,000. Which project should you choose? A. Project A B. Project B C. Project C D. Project D

A. Project A You should always pick the project with the highest Net Present Value (NPV), which in this case is Project A. Since the NPV formula already takes into consideration the time value of money, you can completely ignore the number of years. Also remember if NPV is negative, like Project D, you always reject it.

You have been called into a meeting and are told that you are going to be the replacement for a project manager that was let go recently. You have been told that there is much confusion about the direction of the project, what the high-level goals are, and who is funding the project. There is a list of stakeholders but some of the stakeholders are disputing their involvement on the project. Which document is most likely missing? A. Project Charter B. Stakeholder Register C. Project Management Plan D. Scope Document

A. Project Charter The Project Charter should clearly highlight the high-level goals of the project and identify the Project Sponsor (who is responsible for funding the project).

A medium-sized company is expanding into a new market. However, the company does not have previous experience in that market. A project manager needs to collect requirements. Which tool or technique should the project manager use? A. Prototypes B. Expert judgment C. Product analysis D. Create requirements traceability matrix

A. Prototypes Prototyping is a method of obtaining early feedback on requirements by providing a model of the expected product before actually building it.

As a project manager you notice Stacy and Paul have a disagreement about the technique being used. You notice the conflict is getting out of hand and you decide to focus their attention how well they have performed on scope up until this point. What conflict management technique did you use? A. Smooth/Accommodate B. Withdraw/Avoid C. Collaborate/Problem Solve D. Compromise/Reconcile

A. Smooth/Accommodate Smooth/Accommodate is the technique of emphasizing areas of agreement but ignoring the true problem. By focusing the team members on something different, it could diffuse the situation.

Everyone on your brand new agile team has gone through Jira tool training. Since you have prior experience using the tool, you are mentoring other members of your team who are using it for the first time. Some of your team members are still having difficulty grasping the use of some of the functions and the benefits they provide, since they've never applied them in real-world settings. You share some of your personal experiences with Jira from live projects from your past to help reinforce learning. What kind of knowledge have you shared? A. Tacit knowledge B. Tribal knowledge C. Explicit knowledge D. Experience knowledge

A. Tacit knowledge Tacit knowledge is often difficult to express, usually shared in the form of stories. By explaining examples of past projects using Jira, that is what you have just done in this case. Explicit knowledge is words, pictures, drawings, maps--tangible materials that people can see, feel and touch. Tribal and experience knowledge aren't official terms in the PMI universe for types of knowledge, though they are commonly used terms in various companies and industries.

At what point in a project is the possibility of failure the highest? A. The beginning of the project B. After the initial project plan is published but before work actually begins C. After the halfway point of the project is reached D. Just before the end of the project

A. The beginning of the project Risk and uncertainty are the highest at the beginning of a project. These two factors make the possibility of failure the highest at the beginning. The more you know about a project, the better chance you will have of completing it successfully.

You are an experienced project manager in a fast-paced software development company. You have been assigned to a new project which is different from your previous projects since this project includes a virtual team and utilizing vendors to complete some significant activities. In which of the following cases would you need to consider the contract when preparing the Project Charter? A. The project is being done for an external customer B. You need to procure materials from outside the project C. The project is being done for a new customer D. The project is strategically significant and particularly complicated

A. The project is being done for an external customer Choice A is the correct answer. The contract only becomes relevant to this process when your project is being done for an external party. Thus A is correct.

A large project includes functional groups in four countries. It is close to completion, so the project manager must prepare the closeout documents. How should the project manager obtain appropriate signoff on the acceptance of project deliverables? A. The project sponsor should be asked to sign off. B. The steering committee should be asked to sign off. C. The director of the project manager's business unit should be asked to sign off. D. The four managers from the four countries should be asked to sign off.

A. The project sponsor should be asked to sign off. Since the sponsor provided the resources for the project, it is their responsibility to officially sign off on the project deliverables.

You are participating in a new project charter development for a project with a very tight delivery timeline. You most likely will need to hire additional resources to get this project done on time, which will impact the budget. The sponsor is asking for a general idea on how much this is going cost, in order to expedite budget approval. He would like this estimate to be documented on the project charter. What is your best option? A. Use historical data and provide an analogous estimate, then document that amount on the project charter B. Use historical data and provide a parametric estimate, then document that amount on the project charter C. Insist detailed cost estimates need to be completed before you are willing to provide an estimate for the charter D. Provide an estimate based on your past project management experience, but do not document it on the project charter, as you do not want to be liable for this estimate

A. Use historical data and provide an analogous estimate, then document that amount on the project charter During project charter development, detailed estimating is not known. Due to time constraints and needing an estimated budget up front, analogous estimating should be used. This uses historical data and provides an approximate figure. Parametric estimating (Choice B) involves some type of statistical relationship, which is not mentioned here. Detailed cost estimates (Choice C) will not be done until later in the project process. At this point, we do not even know if project funding will be approved, so this project may never be activated. While past experience may help with the analogous estimating (Choice D), refusing to document is on the charter is not an option since it has been requested by the sponsor.

A pandemic has broken out and the local state officials have banned all public gatherings. You have determined that this will significantly affect the scope of your agile project. What would be a useful tool to help the team function. A. Use remote paring along with your project management information system to keep the team moving forward B. Create a Fishbowl C. Adobe Connect D. Update your communication & resource management plans noting the increased number of channels of communication and the need for cultural awareness training for the team

A. Use remote paring along with your project management information system to keep the team moving forward The Agile practice guide discusses best practices with ""dispersed teams"" and the need for virtual workspace on p 46. Although a fishbowl is discussed in this section the intent of that tool is not to connect individuals with other individuals it's more about connecting a group with another group. The best answer is A using remote pairing.

You are an experienced project manager working in the pharmaceutical industry. You are leading a project for a cutting-edge medication which is revolutionary. You are monitoring the status of the project work to update the project budget and manage changes to the cost baseline. So far things appear to be progressing well, but you are starting to but concerned with work being done by a new vendor. Which output would you be most concerned with? A. Work performance information and cost forecasts B. Budget reserves and cost estimates C. Work performance reports and earned value management D. Cost estimates and aggregation

A. Work performance information and cost forecasts The process described is Control Costs and the outputs are documented in A. This is one of the more technical processes to understand for the PMP exam as it includes the earned value tools and techniques. Budget reserves are monitored during the Monitor Risks process. Earned value management is a technique and not an output. Cost aggregation is when you add all of your individual cost estimates together and is done to determine what the project budget will be.

You are a team facilitator and one of your team members, Jane has approached you with a concerning issue. Another team member, Dave is not following certain guidelines and is violating some ground rules agreed to in the Team Charter. What should you do first? A. Reprimand the team member who is violating these ground rules. B. Refer to the Team Charter to determine what to do in this situation. C. Talk to the team member violating the ground rules and try to understand their perspective. D. Set up a meeting with Jane and Dave to collaboratively try and resolve the problem.

B. Refer to the Team Charter to determine what to do in this situation. The key word is the word first. Ground rules, and what to do if one is not following ground rules is documented in the Team Charter. While options c and d are definitely good approaches since you are using empathy and collaboration respectively, the exact steps on how to resolve such issues will be documented in the Team Charter and that is what you should refer to first. Reprimanding a team member is usually not the best option for PMP questions.

You are trying to determine how best to reward your team for a job well done. Where would you find this information? A. Reward and recognition plan B. Resource management plan C. Stakeholder Engagement Plan D. Cost management plan

B. Resource management plan Rewards and recognition are included in the Staffing Management Plan. Technically the Staffing Management Plan is part of the Human Resource Plan, however, the more precise answer is the Staffing Management Plan. On the PMP exam always choose the more precise answer when asked where certain information would be contained. There is no such thing as a Rewards and Recognition Plan.

You have identified disaster situations (e.g. floods, earthquakes etc.) as potential risks to your project. When discussing with your project sponsor, she mentioned that at the project level, no steps can be taken to deal with such risk and suggested creating a contingency reserve (5% of the project budget) to be used in case of disaster situations. This is an example of: A. Risk Mitigation B. Risk Acceptance C. Risk Avoidance D. Bad policy decision

B. Risk Acceptance Acceptance is a strategy that often must be adopted because it is seldom possible to eliminate all risk from a project. This strategy indicates that the project team has decided not to change the project management plan to deal with a risk, or is unable to identify any other suitable response strategy. This commonly involves "active acceptance," which utilizes a contingency reserve, including amounts of time, money, or resources set aside to help cover a risk event's impact.

You are the project manager for a commercial construction company. Your project involves building a shopping mall. The project must be completed by December 15th or your company will have to pay a weekly penalty of $5,000. This is an example of which of the following? A. Project Requirement B. Schedule constraint C. Assumption D. Cost Constraint

B. Schedule constraint The project must be completed by December 15th, so this is an example of a schedule constraint. Be careful not to confuse requirements with constraints. A deadline is always a constraint. A requirement" is a feature that the deliverable must include or a performance measurement it must meet."

You and the Sponsor are doing a walk through of the construction site and a conversation about Six Sigma arises. She is not a believer in the six sigma methodology, but admits she does not truly know enough to make a true decision. In regards to Six Sigma, what is the best response? A. Lean Six Sigma strives to eliminate defects and increase efficiency B. Six Sigma strives to reduce defects and increase efficiency C. Lean Six Sigma focuses on eliminating six different types of waste D. Six Sigma focuses on eliminating seven differently types of waste

B. Six Sigma strives to reduce defects and increase efficiency Six Sigma strives to reduce defects to no more than 3.4 defects per million opportunities, by eliminating the causes of the defects. This will increase efficiency and productivity within a process. Lean Six Sigma strives to eliminate seven kinds of waste.

Two team members on your regulatory audit project team are in disagreement about ownership on three key deliverables. They have come to you, the project manager, for help because they are unable to come to any kind of resolution on their own. Which of the following conflict resolution techniques is most helpful and least helpful, respectively? A. Compromise, Withdrawal B. Collaboration, Withdrawal C. Collaboration, Forcing D. Smoothing, Withdrawal

B. Collaboration, Withdrawal Collaboration is the most preferred method because it addresses the problem head-on, examines alternatives, and is generally the most thoughtful and proactive approach. Withdrawal would be the least effective option since you essentially remove yourself from the situation. Note, however, that the PMBOK states that any of the techniques may be employed depending on the circumstances.

Project Management consists of 5 Process Groups and 10 Knowledge Areas. Each knowledge area has specific specialization. Of the following, which is not part of Project Integration Management? A. The creation of the project plan B. The integration between project team members C. The execution of the project plan D. The documentation of changes to the project plan

B. The integration between project team members Project Integration Management focuses on tying together all elements of the project plan and ensuring their proper implementation. While B could be considered accurate if the project plan called for some interaction with other project teams, the assumption cannot be made in this instance. A, C, and D are all clearly part of Project Integration Management, so they must be eliminated as answer choices.

Which of the following is true about bid documents? A. They offer no room for bidders to suggest changes B. They ensure receipt of complete proposals C. They inform the performing organization why the bid is being created D. They quote prices as well as the time in which the work can be done

B. They ensure receipt of complete proposals Bid documents ensure receipt of complete proposals by outlining the complete scope of the work at issue and outlining a format to be followed in submitting responses.

You are a project manager for an aerospace company. You manage a project that calls for the production of many special alloy screws which must adhere to very particular standards for use in the aerospace industry. Each screw is measured and weighed, and a notation is made to show value in relation to the mean you have established. What are you using and what is the primary purpose? A. Control Chart, which is a tool and technique of Plan Quality Management, used to track occurrences. B. Control Chart, which is a tool and technique of Control Quality, used to track repetitive activities or results. C. Check sheet, a tool and technique of Control Quality, used to organize facts in a manner that will facilitate the effective collection of useful data about a quality problem. D. Histogram, a tool and technique in Manage Quality, used to show a graphical representation of numerical data.

B. Control Chart, which is a tool and technique of Control Quality, used to track repetitive activities or results. Control charts are used to track repetitive activities or results. Information discovered from a control chart can help determine the stability of a process and whether it would have a predictable performance.

You are the project manager for a start-up software company. In a status meeting, you just learned that a crucial piece of the software your team was creating is not working. This will cause the project to be late by one month and will likely cost additional money for the developers' time. What is the next step you should take? A. Determine the impact of the change to decide whether to initiate Integrated Change Control B. Create a change request to get approval for the revised work required to fix the problem C. Complete root-cause analysis to determine why the problem occurred D. Present the problem to the customer, with your solution to resolve the problem

B. Create a change request to get approval for the revised work required to fix the problem The question states that you have identified the problem and have done the impact assessment (you know that the change will cause a 1 month delay). The next step is to create a change request so that you can get approval to fix the problem. While answer choice D looks good, according to the PMBOK guide you should not go to the customer until after you perform integrated change control.

You are a project manager at a large technology company. This is your first experience managing a technology project and will be relying on subject matter experts throughout your project. In which process are Subject Matter Experts (SMEs) first used? A. Identify Stakeholders B. Develop Project Charter C. Define Scope D. Collect Requirements

B. Develop Project Charter Answer B is correct. Subject Matter Experts are first consulted for their expert judgment in the Develop Project Charter process.

You are a Project Manager in a Multi-Divisional organization. You were just informed by a functional manager that one of your key resources will be unavailable to perform an activity on your Critical Path. What should you do? A. Contact the sponsor immediately B. Discuss the importance of the situation with the manager C. Inform the manager that if they cause project failure, they will regret it D. Update your schedule to account for the resource shortage

B. Discuss the importance of the situation with the manager Discussing the importance of the situation is the best answer in this situation. The authority of a PM is limited in a Multi-Divisional organization, however every effort should be made to develop a solution before contacting the Sponsor or revising the Schedule Baseline. Of the two remaining answers, stressing the importance rather than possible regret is a better course of action in line with Interpersonal and Team Skills.

A project management team member approaches you about the current quality performance. A few of the measurements are captured outside of upper control limits. As a project manager, how do you resolve this issue? A. These are special cause variances and are considered out of control on a control chart B. Identify the root cause of the issue C. These are random causes on a control chart and considered normal D. Create a control chart to track all future changes based on data from the last chart

B. Identify the root cause of the issue Basic rule: if one point falls outside of the upper or lower control limits, the process is out of control on a control chart. Answer A is the appropriate definition of special cause, but does not answer the question of how to resolve it. When something is considered out of control, the first step is to identify the root cause.

You recently took over a project with a team who has been together for three years. You notice the team has multiple conflicts about technical decisions. As the project manager you know this must be dealt with in a proactive manner. With regard to team development, how should you proceed? A. Identify the team is in the storming stage and create team building events B. Identify the team is in the forming stage and create team building events C. Identify the team is in the norming stage and begin execution D. Identify the team is focusing on conflict and using reconciliation techniques to address their issues

B. Identify the team is in the forming stage and create team building events You are a new project manager to the team. The addition of a new team member moves the group back to forming regardless of who the new team member may be. Team building events will help the project manager move the team from forming to norming.

You are facilitating a meeting to discuss a specialized area of your project where your knowledge is lacking. It is vital that a decision be made quickly so the project can proceed. You gather a team of experts in the area to help you make the decision, however, the meeting is not going according to plan. There seems to be much contention between the experts and there are widely-varied opinions on how to proceed. Some members in the meeting are even going so far as making false accusations of others and you notice that the opinion of one member is constantly being rejected. What would be the best approach to try and come up with a resolution? A. Institute a voting method to reach the decision quickly B. Immediately halt this meeting and then use the Delphi technique to reach a decision C. Immediately halt the meeting and engage in team building exercises D. Immediately halt the meeting and set up guidelines for conduct during the meeting, and re-iterating the agenda of the meeting

B. Immediately halt this meeting and then use the Delphi technique to reach a decision The Delphi technique is useful where there is hostility amongst a group of experts, and especially where personal differences are tainting people's reception of others' ideas.

When performing risk analysis, you and your project team realized you might lose a key resource during execution. Your mitigation was to have a non-team member from another department perform her duties. As it happens, the resource did quit and now you must go to the other team member and ask for his time. You know he is now very busy. In order to gain his commitment, you must be very persuasive and clearly articulate your points and positions. This is an interpersonal skill known as what? A. Leadership B. Influencing C. Emotional intelligence D. Negotiation

B. Influencing According to the PMBOK Guide, influencing involves the ability to be persuasive, clearly articulating points and positions, having high levels of active and effective listening skills, awareness of various perspectives and gathering relevant information to address issues.

You are submitting a status report to your external customer. Which communication approach should you use? A. Mass Communication B. Interpersonal communication C. Networks and social computing communication D. Scrum status reporting systems (SSRS)

B. Interpersonal communication Interpersonal communication is the best approach offered as answers to this question. Options A and C are approaches but would not always be appropriate. Option D is a made up term.

Which of the following best describes the primary purpose of the project charter? A. It defines the project scope B. It authorizes work to begin on the project C. It completely describes the work to be done on the project D. It describes the business case and reason for doing the project

B. It authorizes work to begin on the project The main purpose of the project charter is to grant formal authority for work to begin on a project. The charter may include some aspects of the other answer choices, but choice B is the best answer.

You are working with your team and the Product Owner. The product owner is prioritizing the user stories, and the team determines which user stories to include in the next sprint to create a functional deliverable. What does this refer to? A. Minimum Business Increment B. Minimum Viable Product C. Minimum Marketable Feature D. MoSCoW

B. Minimum Viable Product The MVP is the smallest collection of features that can be included to be functional, which is what the scenario is describing. The MMF would be the minimum feature that would be acceptable by the customer. Although the team may deliver a functional product, it may not meet the MMF. MBI is the smallest amount of value that can be added that would benefit the business.

You are a project manager for a large finance company, with more than 100 locations across the country. You are implementing a software product and the finance manager is your project sponsor. You have completed implementation in 50 of the locations to date. You learn the software company vendor has released a significant upgrade on the software you are implementing. When the project began, the functionality was not available, and therefore was not identified as a project deliverable. However, the finance manager did initially request this functionality. What is the most appropriate action for you to take? A. Proceed with your schedule based on the approved project deliverables B. Notify your customer of the upgrade and the associated impacts to the project schedule and then process a change request C. Adjust the project implementation on the remaining sites to include the new functionality as a deliverable D. Honor the original request made by the customer, implement the upgrade to all sites - including the ones already completed then adjust the schedule

B. Notify your customer of the upgrade and the associated impacts to the project schedule and then process a change request Transparency with the project sponsor is very important. The finance manager should be told about the upgrade on the software and the options on implementing the upgrade. The finance manager (Sponsor) should be given the different alternatives in moving forward (including schedule and budget impacts). This should follow the normal change control process.

Which statement best characterizes OPAs regarding how a Project Manager should document lessons learned within their organization? A. Lessons learned are collected throughout the project as way of improving organizational learning B. Policies established for how lessons learned are collected, knowledge management processes and leveraging how prior knowledge is used to improve projects C. Policies established for promoting organizational knowledge, collection, archiving and distribution external to the organization for shareholders D. External influences on the project out of Project Manager control

B. Policies established for how lessons learned are collected, knowledge management processes and leveraging how prior knowledge is used to improve projects The simplest way of understanding OPAs, is knowing they are written down policies and processes internal to the organization. Answer B specifically talks about policies and processes referencing prior knowledge and how Lessons Learned will be collected, managed and used. A is not wrong, but it is not the best answer. C is wrong, because it mentions external shareholders. D is actually about Enterprise Environmental Factors.

You are a project manager on a high visibility education project. You have a great relationship with your sponsor and your charter is detailed and well written. All the following are associated with the Control Quality process except which one? A. Inspection B. Process analysis C. Control Chart D. Fishbone diagram

B. Process analysis Process Analysis is a tool used in the Manage Quality process.

Audits may be conducted to verify a project's compliance with organizational policies, standards, procedures and processes. Which of the following would not be an appropriate auditor resource? A. PMO B. Project Manager C. Internal Audit Team D. 3rd party contractor

B. Project Manager Neither you as the project manager or anyone or your team would be an appropriate choice for the auditor role, as there could be a conflict of interest. Audits should be conducted by other parties outside of your team.

Which of the following steps is not part of conducting a retrospective at the end of a sprint? A. Use a flip chart, white board or similar medium to list what went well and what could be improved for the next sprint. B. Project Manager lists all problems he/she identified during the sprint on the chart or board, and asks the team how it could improve on those items. C. Each participant identifies the reasons for the improvements. D. Add the agreed-upon improvements as tasks on the Product Backlog after discussing with the Product Owner.

B. Project Manager lists all problems he/she identified during the sprint on the chart or board, and asks the team how it could improve on those items. While the Project Manager or Scrum Master may prepare some notes prior to the retrospective to have available in case the team has difficulty with updating the chart or board with observations or ideas, the team primarily does this work. The team then decides on which ones to focus on--they are not directed by the project manager.

You have been assigned as the project manager to make new widgets for the company. Being the proactive PM that you are, you access your corporate knowledge base for historical files for a similar project in the past. You are able to find old files, such as the project charter, project management plan, and archived quality audit reports. Your sponsor requests that you provide her a breakdown of all of the deliverables of the project. The best way to detail that information to your sponsor is to create a WBS. All the following are needed to create the WBS except which one? A. Organizational process assets B. Project charter C. Requirements document D. Project Scope statement

B. Project charter The project charter is not required during creation of the WBS. The project scope statement, requirements documentation, and organizational process assets are the inputs to the Create WBS process.

A market demand, a business need, or legal needs are justifications for which of the following? A. Reasons to have change management B. Reasons to undertake a project C. Reasons to build a team D. Reasons to create a management plan

B. Reasons to undertake a project Market Demand, Strategic Opportunity/Business Need, Customer Request, Technological Advances and Legal Requirements are all reasons to undertake a project.

When should the product features be defined and finalized? A. Before anything else happens in the project B. Before the project management plan is put under version control C. As soon as possible in project planning D. As the executing process group unfolds

C. As soon as possible in project planning The more clarity you have about the details of the project's product, the better you will be at writing the overall plan. While it is true that planning is an iterative process and some enhancement is expected as planning proceeds, the product features should be defined as clearly as possible early on so that the project can be planned and baselined as accurately as possible.

When will stakeholders have the most influence over the outcome of a project? A. At the end of the project B. During scope validation C. At the start of a project D. At the start of each phase

C. At the start of a project Stakeholders have the most influence at the start of the project, when they are providing their requirements which will help shape the scope of the project and its deliverables.

On your project, there are several stakeholders, the most important of whom are within the main business you support. Your sponsor calls you aside one day and advises you he has noticed there appears to be less support and more resistance to your project. What tools and techniques of Manage Stakeholder Engagement should you be using to help deal with this issue? A. Root cause analysis, assumption and constraint analysis, stakeholder engagement assessment matrix B. Stakeholder engagement assessment matrix, active listening, stakeholder analysis C. Conflict management, cultural awareness, negotiation, political awareness D. Stakeholder analysis, stakeholder mapping/representation, questionnaires and surveys

C. Conflict management, cultural awareness, negotiation, political awareness While there is some overlap among the various Stakeholder processes, only Manage Stakeholder Engagement has the four mentioned in the answer. Conflict management in particular only shows up in Manage Stakeholder Engagement, not in the other three.

You are evaluating and recording the results of project deliverables to determine if they meet the quality standards. Which of the following is not part of this process? A. Audit B. Statistical sampling C. Histogram D. Inspection

A. Audit Audit is a structured, independent review to determine whether project activities comply with organizational and project policies, processes, and procedures, and is part of Manage Quality

After the start of a project, the project manager learns that an expected governmental subsidy may not be available. What should the project manager do to address this issue? A. Raise the project risk level. B. Make sure that the information is added to the issue log. C. Make up any project deficit using the contingency reserve. D. Make a change request.

A. Raise the project risk level. The risk level on this project must be elevated as an expected monetary result might not be achieved.

A skincare company reports that customers are increasingly complaining that a product is underweight. A project manager needs to determine if something needs to be changed in the production line. Which tool or technique can the project manager use for this task? A. Scatter diagram B. Affinity diagram C. Control chart D. Histogram

C. Control chart Control charts are used to determine whether or not a process is stable or has predictable performance.

You have been asked to submit a proposal for a project that has been put out for bid. First, you attend the bidder conference to ask questions of the buyers and then clarify questions some of the other bidders asked. Which of the following statements is true? A. Bidder conferences are a tool and technique of the Plan procurement management process. B. Bidder conferences are an input of the Conduct procurements p process. C. Bidder conferences are an output of the Conduct procurements process. D. Bidder conferences are a tool and technique of the Conduct procurements process.

D. Bidder conferences are a tool and technique of the Conduct procurements process. Bidder conferences are a tool and technique of the Conduct procurements process. They are used to ensure that all prospective sellers have a clear and common understanding of the procurement (both technical and contractual requirements), and that no bidders receive preferential treatment

As a project manager, you often use a Responsibility Assignment Matrix (RAM) to show the types of resources required and the responsibility each resource has on the project. In particular, you prefer to develop a RAM that shows the relationship of your resources to the WBS elements of your project. A RAM includes all of the following except for which one? A. Activities or work packages B. Individuals or groups C. A relationship between activities and individuals D. Control Accounts

D. Control Accounts A Responsibility Assignment Matrix (RAM) is a matrix diagram showing the relationship between activities and individuals. This may include activities or work packages, as well as individuals or groups. An example of a RAM is a RACI chart.

Which of the following is not a contracting technique an agile coach can utilize to help create a win-win, collaborative relationship with a vendor? A. Multi-tiered structure B. Emphasize value deliver C. Dynamic cancelation option D. Not-to-exceed time and materials

C. Dynamic cancelation option Option C is correct. Answers A, B, and D are all valid contracting techniques an agile coach would use while negotiating with a vendor. Dynamic cancelation option is not a valid contracting technique.

You know your organization must choose projects that will be the most profitable for the company. This approach to project selection is based on which one of the following? A. Dynamic algorithms B. Opportunity cost C. Economic model D. Pareto diagraming

C. Economic model This is a simple economic model of choosing the most profitable project for the organization. Dynamic algorithms are an example of constrained optimization methods. While this question hints at opportunity cost, this is not the best choice. Opportunity costs describe the scenario when an organization has two or more projects to choose from and they can only do one project because of time or limited resources. Pareto diagraming is not a valid option for this question, as it pertains to quality matters.

You are the project manager for a large manufacturing company. You are currently managing a project to design and manufacture molded fiber and other custom packaging. The inputs to Estimate Activity Durations include which of the following? A. Resource requirements, Resource breakdown structure, Project document updates B. Expert Judgment, Analogous Estimating, Parametric estimating, schedule management plan, risk register, resource, breakdown structure C. Enterprise environmental factors, Organizational process assets, Activity list, Activity attributes, Schedule management plan, resource breakdown structure, Project scope statement, resource requirements, Resource calendar D. Activity Duration estimates, schedule management plan, risk register, resource, breakdown structure and Project document updates

C. Enterprise environmental factors, Organizational process assets, Activity list, Activity attributes, Schedule management plan, resource breakdown structure, Project scope statement, resource requirements, Resource calendar Enterprise environmental factors, Organizational process assets, Activity list, Activity attributes, Schedule management plan, resource breakdown structure, Project scope statement, resource requirements, Resource calendar are all inputs to the Estimate Activity Durations process.

Project team members are spending a lot of time helping maintenance and operations staff with a recently commissioned project component. The project manager needs to validate that the component was signed off and ownership transferred. Which three documents should the project manager use for this decision? A. Final report, organizational process assets updates, and procurement agreements B. Procurement agreements, project document updates, and organizational process assets updates C. Procurement agreements, stakeholder engagement plan, and organizational process assets updates D. Final report, project document updates, and organizational process assets updates

D. Final report, project document updates, and organizational process assets updates When closing the project, the project manager reviews the project management plan to ensure that all project work is completed and that the project has met its objectives. The project documents are updated accordingly.

You are a project manager in the Information Technology department. Your company is new to agile concepts, and while most of your department is familiar with and on board with agile, most of the business units are not. Since key stakeholders will now become part of agile teams, you realize that you will need to mentor and coach them during sprints to help "sell" them on the benefits of agile to help keep them properly engaged. Which of the following would not be a typical benefit of using agile? A. Improved quality from more frequent reviews B. Requirements are allowed to change to better match business needs on a rolling basis C. Greater accountability and productivity of empowered teams D. Increased focus on business value based on prioritization of requirements

C. Greater accountability and productivity of empowered teams While a self organizing empowered team is a crucial part of agile, this is one of the areas that predictive management often struggles with due to a lack of trust or feeling of not being involved the same way as before. Each of the other elements directly ties to business value and can usually be accepted more quickly.

You are the project manager for a health care company. There are 3 phases on your software. You just completed the first phase of your project and are ready for phase 2. Which process groups must you pass through for the second phase? A. Whichever process groups are necessary to meet the objectives of Phase 2 B. Executing and Monitoring & Controlling C. Initiating, Planning, Executing, Monitoring & Controlling, and Closing D. Planning, Executing, and Monitoring & Controlling

C. Initiating, Planning, Executing, Monitoring & Controlling, and Closing The correct answer is choice C. You must follow through all 5 process groups for each and every phase of a project.

You are the project manager for a large medical company. Your project is to create an innovative laser that will dramatically improve how dentists clean teeth. This technology is new and must be manufactured with precision to pass federal regulations and strict inspections. You must have a robust quality management plan. All of the following are tools and techniques of Plan Quality Management except which of the following? A. Expert Judgment B. Data Gathering C. Inspection D. Test and inspection planning

C. Inspection Inspection is a tool and technique used in Control Quality (to ensure conformance with quality standards).

You are managing a project which involves many different stakeholders, with varying interests and expectations from the project. Some are especially cost-conscious, while others are more interested in expediency over cost-savings. Which of the following will be most important for you to employ in order to keep all stakeholders satisfied? A. Earned Value Management B. Rewards and Recognition C. Interpersonal skills D. Team building exercises

C. Interpersonal skills Interpersonal skills, are sometimes called soft skills, and are critical to successful project management. They involve tailoring your approach to suit different personalities and interests among stakeholders. Because you must keep all stakeholders happy despite their differing interests, these skills will be especially important.

In your interviews with some key stakeholders you conclude that the scope statement was poorly constructed. You know all of the following are true except which of the following? A. It will be difficult to assess future project decisions from this scope statement. B. It will be difficult to decompose the deliverables from this scope statement. C. It will be difficult to identify all stakeholders D. It will be difficult to create an accurate WBS from this scope statement.

C. It will be difficult to identify all stakeholders Stakeholder identification occurs during the initiation process group, and a failure in the subsequent development of the scope should not affect the identification of stakeholders. All of the other answer choices are correct regarding what can happen if you have a poorly defined scope statement.

You are the project manager for a large education company. You have had some troubles in the past keeping team members motivated on other projects. You want to make sure your team on the current project is highly motivated, since this is an especially important and complex project. Which of the following is the best method to make the reward systems more effective? A. Pay a large salary increase to the best workers B. Give the team a choice of rewards C. Make the link between performance and rewards clear D. Present notification of rewards within the company

C. Make the link between performance and rewards clear The best method to make a reward system more effective is to clearly link performance to the reward, and communicate this to the team member so they know that certain performance will be rewarded.

You are the project manager for a large Education company. You are currently managing a project to handle all aspects of marketing communications for higher education clients, from strategy to campaign. This is a new idea and a new product. A new product will be created based on market research. This is an example of which of the following reasons to launch a new project? A. Organizational need B. Customer request C. Market demand D. Legal requirement

C. Market demand Projects can be created for a number of reasons, and this example supports the market demand choice.

Which of the following is the purpose of a retrospective? A. To determine if the deliverables meet the definition of done. B. To close out the sprint. C. To allow the Product Owner to accept the deliverable. D. For Stakeholders to reward the team.

B. To close out the sprint. A retrospective is the last Agile ceremony on a sprint whose primary purpose is to discuss lessons learnt and close out the sprint. This occurs after the sprint review meeting where the Product Owner (not the team) approves based on the definition of done and accept the deliverable. The purpose is not to reward team members at this meeting.

You are a project manager for a new medical device. Current market demand requires the new device to be capable of being used in various conditions, by differing levels of user expertise, be made available in different languages all over the world, and other external factors. Which of the following would best be utilized to analyze or identify risks for all of these factors? A. Risk register B. Stakeholder register C. PESTLE Analysis D. Probability and Impact Analysis

C. PESTLE Analysis Of the options presented, a PESTLE analysis is used to identify the external business environment factors that can affect the value and desired outcomes of a project.

Who decides what work gets done in an agile team? A. The project sponsor B. Key stakeholders C. The agile team as a whole, which is self-managing D. The project manager

C. The agile team as a whole, which is self-managing Answer C is the most inclusive. The team should be self-managing.

Anything done to bring expected future schedule performance in line with the project management plan is known as ________. A. Reduction in Scope B. Corrective action C. Preventive action D. Re-baselining

C. Preventive action The PMBOK Guide definition of preventive action is any action taken to bring anticipated future project performance in line with the project management plan.

The features and functions of a product or service are considered ________, and the work that must be done to deliver a product/service as the output of the project is considered ________. A. Project scope, Product scope B. Requirements, Deliverables C. Product scope, Project scope D. Scope management plan, Project management plan

C. Product scope, Project scope The correct answer is C. Product scope describes the features and functions of the product or service, whereas project scope describes the work that must be done to deliver a product/service as the output of the project.

You are a project manager for the state. You are managing a virtual team with team members dispersed throughout the state. You are preparing a document which outlines high-level requirements, known high-level risks, a summary budget, and a description of the project manager's level of authority. What knowledge area is being utilized here? A. Project Scope Management B. Project Stakeholder Management C. Project Integration Management D. Project Communications Management

C. Project Integration Management Project Integration Management, which focuses on the coordination of all components of project management, includes the development of the project charter (which is what the question describes).

On agile projects, the product owner has several duties and responsibilities. Chief among those duties is ownership of the product backlog. Which of the following additional duties is not the responsibility of the product owner? A. Responsible for defining and prioritizing the user stories with the help of the team. B. Answers team questions about the needed solution. C. Provides relative sizing estimates for the user stories in the product backlog. D. Accountable for the ultimate business value of the solution produced by the project team.

C. Provides relative sizing estimates for the user stories in the product backlog. The agile team is responsible for determining the relative estimates for agile projects. The product owner may or may not participate in that process.

When you and your project team performed risk analysis, one of the identified risks was the material you needed, while less expensive, was not in plentiful supply. Your risk response was to import materials from overseas, as meeting the schedule was deemed more important than meeting the budget. During project execution, an expected shortage did in fact occur and you had to implement the risk response. Your next move should be to do what? A. Extend the schedule B. Start looking for another supplier C. Put in a request to change the cost baseline D. Escalate to the sponsor

C. Put in a request to change the cost baseline You may do any of the other items but the first thing which must be done is to put in a change request to change the cost baseline as the materials are now raising your costs.

Which of the following is not a decision-making technique for agile projects? A. Fist of five B. Roman voting C. Dot voting D. Planning poker

D. Planning poker Planning poker is a technique used to relative estimate user stories. The others are decision-making techniques.

You are a project manager producing several identical components for an aerospace project. You are supposed to be producing 2 units per month worth $250,000 per unit. After 3 months you have only produced an average of 1.2 units per month. You use a formula to determine what the difference is between what you planned to do and what you actually got done. What is the formula you used? A. Schedule Performance Index (SPI) B. Cost Variance (CV) C. Schedule Variance (SV) D. Variance at completion (VAC)

C. Schedule Variance (SV) The question is talking about calculating a schedule variance (SV), which is the difference between what you produced (Earned Value) and what you were supposed to complete (Planned Value).

Agile is built upon communications, including feedback loops. Which of the following would least be considered a feedback loop? A. Pair programming B. Daily scrum meeting C. Sprint D. Retrospective

C. Sprint Option C is correct. Pair programming, the daily scrum meeting, and retrospectives are examples of feedback loops. Information is exchanged and allows changes to be made if required. The sprint is the timebox where the agreed-upon work to obtain the sprint goal is performed.

You are in charge of the PMO at a consulting services company. You are creating a lunch-and-learn training session on Procurement, and it will include a section on contract negotiations. You want to make sure your junior project managers understand contract negotiations and the role that they play. For your content, what is the most important thing to focus on in contract negotiations? A. To negotiate the best price possible for your project. B. To maintain the integrity of the scope. C. To negotiate a deal that both parties are comfortable with. D. To make sure legal counsel or the contract administrator has approved your negotiating points.

C. To negotiate a deal that both parties are comfortable with. The correct answer according to the PMBOK Guide is C. The most important thing to focus on is to create a deal that everyone feels good about. Choice A sounds like a good choice, but the best possible price might not be fair to your seller, and that could create a bad scenario for the project in the future. Choice B is important, but that is not the primary focus of negotiations. Choice D may or may not be necessary, depending on the situation.

You are the Project Manager for ACME software company. Your current project is doing well, but you have many defects. You submitted a change request that recently came back approved. You have had several issues in the past with change requests not fulfilling their requirements. As the Project Manager, how should you proceed? A. Update the change log, implement changes, and monitor and control all changes B. Implement the change and hope it will be different this time C. Update change log and other documents, inform the stakeholders, and follow the new plan D. Conduct configuration management and direct project work

C. Update change log and other documents, inform the stakeholders, and follow the new plan The question is truly asking about the steps in change control. The question states a change request recently came back approved. Do not be distracted by the scenario. Steps in Change Control Process are: 1) Prevent the root cause of changes, 2) Identify the change with documentation, 3) Evaluate the impact of the change, 4) Issue the change request to the CCB, 5) Perform Integrated Change Control (get CCB approval), 6) Update change request log and project documents to reflect the change, 7) Communicate the change (and impacts) to stakeholders and get buy-in, and 8) Direct and manage the project according to the new plan.

You are the Project Manager for an organization that embraces changes to the business environment. More than likely what type of organization do you work for? A. Functional B. Weak Matrix C. Tight Matrix D. Projectized

D. Projectized Most likely this would describe Projectized. That is not to say that Functional or Weak Matrix do not embrace change. Both Functional and Weak Matrix normally refer to ongoing operations type work with few projects, whereas Projectized organizations only work on projects. Projects are all about change and so that would be the best answer. Remember you will not always get perfect answers (or perfect questions for that matter), you have to choose the best out of an ambiguous situation. Tight matrix refers to the team being co-located (not a type of organization).

Projects are dynamic and include many moving parts and stakeholders. As a project manager you should know how various factors influence your project. Which of the following statements is true? A. Cost and Staffing level are low at the beginning of the project and continue the same trend until closure. B. Stakeholders influences, risks, and uncertainty are greatest at the end of the project. C. Cost of changes is high at the beginning of the project and low at the end of the project. D. Stakeholders influences and risks are highest at the start of the project

D. Stakeholders influences and risks are highest at the start of the project Stakeholders influences and risks (uncertainty) are highest at the start of the project. These factors decrease over the life of the project. Changes are also more expensive as time passes on a project.

What can be used to improve the accuracy and reliability of release planning? A. Consensus building B. Affinity estimating C. Sprint planning meeting D. Velocity

D. Velocity Option D is correct. By understanding a teams current velocity (the number of story points a team can finish per sprint based on past performance) and knowing the remaining story points required for the release, a release date can be forecasted more accurately.

You are conducting risk management activities on your project. You have gathered pertinent stakeholders together and identified all risks. Next, you analyze the risks to determine an appropriate amount of contingency reserves to put into the project baselines. What did you forget to do? A. You need to Plan Risk Responses before doing determining contingency reserves B. You need to Identify Stakeholders to ensure you have identified all stakeholders needing to be involved in this process C. You need to Perform Quantitative Risk Analysis to prioritize the risks before Perform Qualitative Risk Analysis, which you use to identify contingency reserve amounts D. You need to Perform Qualitative Risk Analysis to prioritize the risks before Perform Quantitative Risk Analysis, which you may use to identify contingency reserve amounts

D. You need to Perform Qualitative Risk Analysis to prioritize the risks before Perform Quantitative Risk Analysis, which you may use to identify contingency reserve amounts Choice D is correct as you need to prioritize the risks (Qualitative Risk Analysis) before you quantify the risks (Quantitative Analysis). Prioritization will result in the risks being prioritized to determine which risks are significant. The risks which are relatively insignificant on probability and impact will be put on a watch list and will not be reviewed quantitatively. Choice A is incorrect as Perform Quantitative Risk Analysis may be used to calculate contingency reserves and results from this process are used as inputs to the Plan Risk Responses process. Choice B is incorrect since this question is not addressing stakeholder input, but rather a step that you missed. Choice C is incorrect as the terms are mismatched.

What is the ideal size of an effective agile team? A. 3 - 9 members B. 10 - 12 members C. 4 - 5 members D. Less than 12 members

A. 3 - 9 members In practice, the most effective team size is 3 - 9 members. Answer D is partly true but A is the best answer.

For a software development project, which project life cycle/methodology lends itself best to a mutual understanding of requirements between customers/end users and the project team? A. Agile B. Six Sigma C. Waterfall D. Iterative

A. Agile When working on a software project, agile is the most often preferred life cycle approach. This allows the team autonomy to self-create and direct, as well as work directly with the customer or end user on the final product outcome.

A project was not meeting scheduled dates for some tasks, so the project manager reprioritized a number of subsequent tasks to try to get the project back on track. The reprioritization did not fix the problem and now the project manager needs to escalate the issue to project stakeholders. To escalate the issue, to which plan should the project manager refer? A. Communications management plan B. Stakeholder engagement plan C. Change management plan D. Baselines for scope plan

A. Communications management plan The communications management plan is developed to ensure that the appropriate messages are communicated to stakeholders in various formats and various means as defined by the communication strategy.

You are leading a project to create a degree plan for a regionally accredited university. The new degree is highly specialized and requires the approval of curriculum by governing body of electrical engineers. The program already has students enrolled, but they will not start until the fall which is three months away. If the program fails to meet the standards mandated by the Society of Electrical Engineers it would be public relations nightmare. What should the project manager do to ensure all requirements are met? A. Create a tailored quality management process to any specific quality standards and regulatory constraints B. Create a policy and based on Organizational Process Assets (OPA) and conduct audits C. Enter into a contract with the engineers to oversee all compliance requirements D. Seek out insurance to hedge against financial liability and bring a PR agency on board in case of failure.

A. Create a tailored quality management process to any specific quality standards and regulatory constraints Tailoring considerations include standards and regulatory compliance to quality standards, governmental, legal, or regulatory constraints that must be considered.

Which of the following would be least useful in establishing a strong project vision? A. Creating a highly-detailed vision that focuses on what will be done B. Creating a vision that is customer focused C. Frequently discussing the product vision D. Ensure the product vision aligns with the organization's overall vision and strategy

A. Creating a highly-detailed vision that focuses on what will be done Option A is correct. A highly-detailed and complex vision is difficult to remember and may confuse team members. A short, concise project vision offers clarity. Answers B, C, and D would each assist in building a strong project vision.

A project manager has been asked to conduct a risk analysis that is based on a high-level scope. As part of the analysis, the project manager must use expert judgment to prepare a document. Which document-based action is being performed? A. Creating the project charter B. Preparing the scope statement document C. Creating a project management plan D. Documenting the risk management plan

A. Creating the project charter Expert judgment is one of the tools and techniques for developing the project charter.

A project is completed, but the project manager and project team members are being asked to do the following: Troubleshoot and resolve technical issues, evaluate new requirements, make minor adjustments. The project manager needs to ensure that the product developed by the project now has operational support. What should the project manager do? A. Execute the project hand-off. B. Procure project acceptance. C. Contact the project team, dismiss them from the project, and find a new project. D. Since the product is still under development, continue to fulfill the requests.

A. Execute the project hand-off. At project close, the final product, service, or result can be transitioned to a production support team.

You recently took over a team from a project manager who submitted their two week resignation in lieu of being terminated. You notice the team lacks trust and commitment as well as harboring resentment issues. As the project manager, what is the first thing you should do? A. Hold a collaboration meeting with the team to discuss the team vision B. Conduct brainstorming sessions with the team to identify causes of dysfunction C. Define the team vision, then brainstorm the root cause of any issues D. Review the team charter and document on the risk register

A. Hold a collaboration meeting with the team to discuss the team vision The first step is to discuss the team vision and build a consensus with all team members. Once complete, brainstorming the root cause of the issues can proceed. Defining the team vision for the team does not create buy-in, this must be done with the team. Annotating it on the risk register would not be your first step.

A company wants to reduce costs by installing solar panels. Their local government provides subsidies for specific energy-efficient projects. After project start, the program manager learns that solar panels may not qualify for the energy-efficiency subsidy. How should the program manger address this issue? A. Increase the risk level for the project. B. Ensure the issue log is updated. C. Make use of the contingency reserve. D. File a change request.

A. Increase the risk level for the project. The risk level on this issue must be elevated as an expected monetary result might not be achieved.

What is the main purpose of the work breakdown structure (WBS) dictionary? A. It contains additional information about each work breakdown structure (WBS) component B. It provides a framework for how to create the work breakdown structure (WBS) C. It tracks time and cost on a project D. It contains additional information about each activity

A. It contains additional information about each work breakdown structure (WBS) component The main purpose of the work breakdown structure (WBS) dictionary is to store additional information about each work breakdown structure (WBS) component. This can include things like work package name, ID, responsible organization, schedule milestones, quality requirements, cost estimates, required resources, code of accounts identifier, etc.

The project is overseen by a senior project manager. When a new project manager is added to the project, the sponsor asks the new project manager to develop the quality management plan. How should the new project manager proceed? A. Obtain established quality processes and plans from the senior project manager, then ask them for guidance in selecting the appropriate standards. B. Schedule a meeting with the customer to collect requirements and obtain benchmarking samples. C. Schedule a meeting with the sponsor to obtain benchmarking samples and cost-benefit analyses. D. Postpone the development of the quality management plan by submitting a change request. Only proceed when quality metrics and quality checklists have been created.

A. Obtain established quality processes and plans from the senior project manager, then ask them for guidance in selecting the appropriate standards. The new project manager needs to consult with the senior project manager to find out what practices and processes are already in place.

Which of the following eXtreme programming (XP) practices best assists with changes affecting project scope? A. On-site customer B. Planning games C. Simple design D. Coding standards

A. On-site customer Option A is correct. The on-site customer practice includes embedding a customer (or customer representative) into the team itself. Therefore, if external circumstances, (market conditions, competition, financial considerations, etc...) present themselves, the agile team can act quickly and decisively.

A project has the following characteristics: It is in the planning stage; A key stakeholder will be unavailable for 2.5 weeks during the project. The project manager must decide what to do first, after learning about the key stakeholder's unavailability. What is the correct step to take? A. Plan a database development specification, then occasionally check to ensure the developer aligns with the specification. B. Schedule quarterly project review meetings that will include review of the data security status, and recommend remedial steps if noncompliance is found. C. Organize a meeting with the sponsor to evaluate the situation and identify alternatives. D. Continue with planning and acknowledge the impact the change may have on the project schedule.

A. Plan a database development specification, then occasionally check to ensure the developer aligns with the specification. The first thing the project manager needs to do is to capture this as an issue and communicate this to the other stakeholders. Until then, no action can or should be taken.

A project manager joins a project that has the following features: -Globally distributed team -Budget that is fixed -Clearly defined scope -Very tight deadlines. During planning, what should the project manager do to avoid any scope creep and keep the team focused on delivering a product that meets requirements? A. Put any changes to scope through the formal approval process. B. Hold daily stand-ups with all development teams. C. Escalate all changes to scope to upper management. D. Disallow stakeholders from directly interacting with the development team leads.

A. Put any changes to scope through the formal approval process. Scope creep can be avoided by adherence to the agreed upon change management process.

After six iterations, significant variations have occurred to the project scope and schedule. These are due to the customer's technology regulations and security policies, which are regularly reviewed. The project manager wants to suggest a new approach to the next iteration to avoid possible delays. What should the project manager do first? A. Rank the importance of each improvement item, then implement actions identified during the last retrospective. B. Engage in a cost-benefit analysis that can help predict the profits that could be realized through the use of new technology for the next phase. C. Research new technology trends and management tools that have been successful on similar projects. D. Use a backlog refinement meeting and include the customer's change requests in the discussion.

A. Rank the importance of each improvement item, then implement actions identified during the last retrospective. The retrospective is used to review lessons learned and suggest corrective actions for the next iterations.

A project that is nearing completion has the following characteristics: It is very large, it contains functional groups in five different countries, closeout documents are being prepared, the project manager needs to gain acceptance of the project deliverables. What should the project manager do? A. Seek sign off from the project sponsor. B. Contact the steering committee for sign off. C. Seek sign off from the director of the project manager's business unit. D. Contact managers from the five countries involved in the project for sign off.

A. Seek sign off from the project sponsor. The project charter determines who will sign off on the project, but of the given options, the sponsor is the best answer due to their role.

Today you sign into your teams platform to get up to speed on what the Asia-Pacific team has been up to since you signed off last night to enjoy an evening with the family. As you are scrolling through the conversation string you notice that a few comments came up that seem like one of your team members has taken offense to one of the comments left by one of the North American team members from yesterday. Which of Maslow's hierarchy of needs involves the need for friends, family and affiliation? Also, what is the level just below that? A. Social, Safety B. Social, Esteem C. Cultural, Social D. Cultural, Safety

A. Social, Safety The rung on Maslow's pyramid that is marked by an individual's need for friends, family, and affiliation is the Social level. The level just below that is Safety. The five levels in order from bottom to top are physiological, safety, social, esteem and self-actualization; cultural is not part of Maslow's pyramid.

During the design phase of a construction project, two team members do not agree with an estimating technique used for cost. The situation was not resolved and has escalated into a bigger problem that is beginning to impact project objectives. How could this problem have been avoided? A. The project manager should have documented the problem on the issue log B. The project manager should have documented this on the risk register C. The project manager should have escalated the problem to the sponsor for resolution D. The project manager should have submitted a change request to update the cost management plan

A. The project manager should have documented the problem on the issue log Since this situation indicates something has already happened - it is an issue and not a risk. The issue log will help the project manager effectively track and manage issues, ensuring that they are investigated and resolved. The risk register is for events that pose a threat to the project, but have not yet occurred, and the team proactively attempts to manage them.

After a project delay, the project manager determines that a change is needed to the product deliverables in order to ensure quality. The project manager takes the following steps: 1. A formal change request is issued. 2. The project manager communicates the change to all stakeholders. 3. The project manager receives feedback that the communications approach did not meet stakeholder expectations. What should the project manager do next? A. Use the communications management and stakeholder engagement plans. B. Ensure stakeholders' expectations are included in the project management plan by issuing a change request. C. Hold a meeting with the project sponsor to better understand stakeholders' expectations. D. Refer to the communications management plan and issue a status report.

A. Use the communications management and stakeholder engagement plans. These two plans help establish how communication will be managed and how the stakeholders will be engaged.

The procurement department has been negotiating the terms of a new contract with a customer who is unsure of what the final product should look like. They only know what the product should do once complete. The procurement department is having difficulty choosing between a fixed price contract or a cost reimbursable contract and they decide to call you, the project manager, in to assist with negotiations. As the project manager, how should you proceed? A. Work with the customer to determine the objectives and clarify requirements B. Elect to use a cost-reimbursable contract with a Point of Total Assumption Clause C. In a centralized purchasing organization the project manager is responsible for negotiating the contract D. Provide a bottom up estimate to determine a baseline cost and hold a bidder conference

A. Work with the customer to determine the objectives and clarify requirements Customer collaboration over contract negotiation is the preferred method. Since you have a procurement department, the project manager's role is to provide information about the aspects of the project itself, but they do not conduct contract negotiation.

While reviewing the issues log, the project manager notices that tasks assigned to two team members are quite a bit behind schedule. What should be done? A. Work with the relevant team members to review the delayed tasks and decide on strategies for resolution. B. Document the issue, escalate to the project sponsor, and ask how they would like it resolved. C. Tell the team members who are behind schedule that they should explain why tasks were delayed and how they will get back on schedule. D. Notify the responsible team members of the need to complete the overdue tasks quickly in order to avoid introducing extra project risks.

A. Work with the relevant team members to review the delayed tasks and decide on strategies for resolution. The project manager needs to be proactive and review the activities with the team and to identify resolution strategies.

You are a project manager for a large aerospace company. You are working on manufacturing a space shuttle turbine. Design for X performs which of the following functions? A. identifies which variables have the most influence on a quality outcome B. identifies which variables have the least influence on a quality outcome C. determines what a quality outcome is D. identifies variables impacting quality outcomes by looking at each variable individually

A. identifies which variables have the most influence on a quality outcome Design for X is a statistical method of identifying which factors may influence specific variables of a product or process under development. Design for X should be used during the plan quality management process to determine the number and type of tests and their impact on cost of quality. D is incorrect because Design for X analyzes all variables collectively, not individually.

When working in a highly regulated industry, what can be done to ensure any changes in regulatory compliance are met during an agile project? A. Evaluate regulatory compliance during the daily scrum B. Continuously groom the product backlog C. Use affinity estimating D. Refactoring

B Continuously groom the product backlog Requirements for new and/or changing regulatory compliance will be incorporated into the product backlog as they arise if the product owner grooms (or refines) the backlog regularly.

A project manager is uncertain about the duration of a new product development project and consults several groups of subject matter experts (SMEs). They advise that the fastest development could possibly be finished is in 8 days. However, they have also identified some risks that could make the duration up to 32 days. Realistically, using standard development methods, the project should take 14 days. The team agrees that this final estimate is the most likely to occur and should be weighted by 4x accordingly. What is the estimated development duration for the new product? A. 12 B. 16 C. 18 D. 20

B. 16 Using the Program Evaluation and Review Technique (PERT) method of three-point estimating, the formula is (Optimistic + (4 X Most Likely) + Pessimistic)/6 = (8 + (4*14) + 32)/6 = 16 days.

As the new project manager for the Better Beach Bicycle Company, you and the project team are reviewing the work breakdown structure (WBS) which was created by the previous project team. After the project was postponed indefinitely, the previous project manager and project team were released. You notice that the lowest level nodes of the work breakdown structure (WBS) do not include a list of activities. Which of the following are true? 1 - You realize that the previous project team did not finish the work breakdown structure (WBS) since it should have included the required activities. 2 - This is OK since the lowest level of the work breakdown structure (WBS) should be the work package level. 3 - You realize that the previous project team may have captured the required activities in another document and you should keep reviewing their work. 4 - You realize that the previous project team did not finish the work breakdown structure (WBS) since it should have included the required resources. 5 - This is OK since the previous team was using rolling-wave planning, therefore the lowest level of the work breakdown structure (WBS) is the work package level. A. 1 and 3 B. 2 and 3 C. 2 and 4 D. 4 and 5

B. 2 and 3 A work breakdown structure (WBS) is a hierarchical decomposition down to the work package level. A work breakdown structure (WBS) never includes resources or activities lists. Rolling-wave planning can be done after completing the work breakdown structure (WBS) but is not necessarily used to create one. There is also no indication in the question that the previous team used rolling-wave planning.

Which of the following would be the team facilitator's least concern when building an agile team? 1 - Whether or not team skills are I-shaped or T-shaped 2 - The level and amount of agile experience 3 - If the team members can commit 100% of their time to the project 4 - That all the team members are agile certified 5 - Whether or not a stable work environment can be created A. 1 and 4 B. 2 and 4 C. 2 and 5 D. 3 and 5

B. 2 and 4 Option B is correct - the level and amount of agile experience is not something a team facilitator would concern themselves over the other three answers. Successful agile teams have dedicated personnel, are made up of a team of generalists and specialists, should be co-located for better knowledge sharing and improved team dynamics, and should have a stable work environment.

A project manager needs to ensure that all data security requirements are covered during a data warehouse development project's execution stage. Each of the databases created during different project phases will have a specific data security policy to comply with governmental data security laws. How should the project manager plan to meet these requirements? A. Plan a database development specification, then regularly check to ensure the developer aligns with the specification. B. Arrange for review of the data security status on a quarterly basis, and recommend remedial steps if noncompliance is found. C. Tell the database developer to submit documentation of data security items and the steps they take to correct items that do not meet specifications. D. Add a quality assurance specialist resource to the project team, who will be responsible for monitoring the data security quality of the project.

B. Arrange for review of the data security status on a quarterly basis, and recommend remedial steps if noncompliance is found. The project review is the best opportunity to formally review compliance. These reviews should be scheduled quarterly or at significant milestones.

You have just been informed that you have been selected as the new project manager for the XYZ Widget project, which is on a very tight schedule. You recall that previous projects for this customer did not go as smoothly as planned with respect to both the budget and schedule so there is a lot of pressure to meet expectations on this new project. Which of the following should you do first? A. Schedule a kick-off meeting to begin brainstorming with your project team to review the schedule and to understand any issues or concerns that may immediately arise B. Ask for a copy of the project charter so you can review who the major stakeholders are, what the high level budget, schedule, and requirements are and verify your authority as the project manager C. Immediately begin identifying requirements in order to get a jump on the schedule since your primary concern is ensuring that the expectations of the project and the customer are fully met this time D. Review the project scope statement to ensure you fully understand all of the requirements and criteria for success and then schedule a meeting to begin identifying stakeholders

B. Ask for a copy of the project charter so you can review who the major stakeholders are, what the high level budget, schedule, and requirements are and verify your authority as the project manager A project is not officially authorized until the project charter has been created, so no other activities should be started until it has been created. Since you have already been identified as the new project manager, the project charter has already been created. The scope statement would not be created until after you have defined all of the requirements.

You are the project manager for a large construction company. You are currently managing a project to expand an existing facility. Which of the following is an output of Control Scope? A. Workarounds B. Change requests C. Transference D. Risk assessment

B. Change requests Change request are outputs of Control scope. Choices A, C, and D are all related to risk.

While completing your schedule model maintenance a team member informs you a stakeholder from another department has stopped sending updates, showing up to meetings, and has begun speaking negatively about the project openly. Based on the stakeholder's position and responsibilities, it is beginning to take a heavy toll on the project. What could the project manager have done to prevent this? A. Documented this risk in the risk register along with potential responses B. Conducted a stakeholder analysis and monitored stakeholder engagement more closely C. Created a communication management plan that would alert them sooner when a stakeholder was becoming resistant D. Pulled the stakeholder aside and mentor them

B. Conducted a stakeholder analysis and monitored stakeholder engagement more closely A stakeholder analysis would have identified their power, interest, and/or influence on the project allowing the project manager to determine the proper engagement level. Monitoring the stakeholder's engagement would have increased the efficiency and effectiveness of the engagement strategies, which are generally determined, in part, by the engagement level of stakeholders. Documenting the stakeholder on the risk register would be more reactionary. The communication management plan is the wrong document use here.

Which of the following is not considered a servant leader responsibility? A. Educating stakeholders on the benefits of servant leadership and the benefits of agile B. Creating the project development approach and processes C. Providing coaching, mentoring, encouragement and support D. Assisting with technical project management activities

B. Creating the project development approach and processes Option B is correct. Servant leadership does not encompass creating the project development approach or processes. However, a few of the responsibilities of a servant leader are: (1) educating stakeholders on the benefits of agile; (2) provide coaching, mentoring, and support; and (3) assisting by performing the technical aspects of project management.

A project manager realizes that sponsor expectations are not being met, specifically regarding project deliverables. To address this issue, which action should the project manager take? A. Create a risk register. B. Develop a stakeholder engagement plan. C. Develop a communications management plan. D. Create a work breakdown structure (WBS).

B. Develop a stakeholder engagement plan The stakeholder engagement plan is a component of the project management plan that identifies the strategies and actions required to promote the productive involvement of stakeholders in decision making and execution.

For organizations that are new to agile concepts and are working to adopt and utilize them, which of the following would be a potential roadblock to successfully achieving this? A. Executive management's willingness to change. B. Employees are specialized contributors. C. Talent management maturity and capabilities. D. Focus on short-term budgeting and metrics versus long-term goals.

B. Employees are specialized contributors. Organizations that have change-friendly characteristics of answers A, C and D will easily support agile principals. While answer B (indicating I-shaped people) may be a concept that we shy away from, it has nothing to do with organizations adopting agile approaches.

A customer is contemplating adding new features to the project deliverable. Before a project team member starts working on these new features, what is a mandatory step? A. Gain approval from the project manager. B. Ensure the change control board (CCB) approves. C. Gain approval from the project sponsor. D. Ensure key subject matter experts (SMEs) approve.

B. Ensure the change control board (CCB) approves. The change control board (CCB) is a formally chartered group responsible for reviewing, evaluating, approving, deferring, or rejecting changes to the project and for recording and communicating such decisions.

Review the following project characteristics: -The project is in its initiation stage. -Due to lack of funding, the project gets terminated. -The project manager must close the project. What is the first action the project manager should take? A. Rely on the communications management plan to properly inform all stakeholders. B. Examine the organization's project closure guidelines. C. Determine the potential impact of terminating the project by performing a risk analysis. D. Close all project procurement processes.

B. Examine the organization's project closure guidelines. Project closure guidelines or requirements will be contained in the organization's procedures.

You are a project manager of a project close to completing the delivery of the final product when an important stakeholder contacts you about a requirement that seems to be missing from the project scope. You schedule a meeting with them to discuss the specifics of the requirement and the possible options for handling it. During the meeting, the conversation becomes somewhat heated when the stakeholder demands to know when the requirement will be completed. What is the ideal way to handle this situation? A. Use the conflict resolution technique of Smoothing to examine the alternatives and identify a win-win solution B. Explain that any details on whether the requirement is approved or might be completed would be made by the Change Control Board (CCB) and assist them in completing a change request. C. Recognize the stakeholder as a resistor and leave the meeting room to avoid further conflict with a stakeholder D. Agree to personally handle their request to ensure that it is completed as soon as possible to avoid any further conflict

B. Explain that any details on whether the requirement is approved or might be completed would be made by the Change Control Board (CCB) and assist them in completing a change request. The Change Control Board (CCB) must review and approve all change requests. Explaining this fact and then assisting with the completion of the change request is the best way to handle this. The technique of smoothing does not involve examining alternatives and is not win-win. Withdrawing during a conflict is never the best option. Agreeing to personally handle the request and to ensure it's completed as soon as possible sets an unrealistic expectation that it is already approved.

A new product is released. When a customer identifies performance issues with this product, the project manager realizes that cost of quality (COQ) should have been used to estimate this cost. Which category of COQ should the project manager have used? A. Prevention costs B. External failure costs C. Appraisal costs D. Cost-benefit analysis

B. External failure costs External failure costs are those failures found by the customer.

You are the project manager in a service industry organization. You are leading a project to implement a new Customer Relationship Management (CRM) system with a key client. You have just received your Project Management Professional (PMP) certification and remember stakeholder influence is greatest at the beginning of the project. You have just met with the sponsor and received your formal authorization to start the project. You want to make a good first impression so you are focusing your attention conducting a thorough stakeholder identification process. What should you focus on and what would not necessarily be valuable at this time? A. Focus on the project management plan to identify stakeholders; the project charter probably is not of value. B. Focus on the Business case; the communications and stakeholder engagement plans are probably not of much value. C. Focus on the project charter; the business case and benefits management plan are probably not of value. D. Focus on the benefits management plan; the communications and stakeholder engagement plans are of most value.

B. Focus on the Business case; the communications and stakeholder engagement plans are probably not of much value. The key is you are at the beginning of the project. At this time your business case is a great resource to identify stakeholders. The communications management plan and stakeholder engagement plan are not available when initially identifying stakeholders.

What is the term you would use to describe the amount of time an activity can be delayed without delaying the successor activities? A. Critical path method B. Free float C. Total float D. Lag

B. Free float Free float is the amount of time an activity can be delayed without delaying the earliest start of a successor activity. Total float is the amount of time an activity can be delayed without delaying the entire project.

You are the project manager of a large global project that has team members spread out across several countries in multiple time zones. Several of the team members in India have been complaining about the limited health benefits provided by your company and that they are not able to pay for the care they need, given their lower salaries as compared to project team members in the United States. Lately, they have been somewhat slow to respond to your e-mail requests for status updates and have even missed a few meetings. Which of the following is the most likely explanation for this? A. McGregor's Theory-X B. Herzberg's Theory of Motivation C. McGregor's Theory-Y D. McClelland's Theory of Needs

B. Herzberg's Theory of Motivation Herzberg theorized pay and benefits do not lead to positive satisfaction for the long-term. But if these factors are absent or non-existent at the workplace, then they lead to dissatisfaction. McGregor's Theory-X and Theory-Y theorize that employees inherently dislike work and are self-motivated, respectively. McClelland's Theory of Needs says people are motivated by either 1) achievement, 2) affiliation, or 3) power.

Who is responsible for orchestrating and integrating all planning activities for an agile project? A. Project sponsor B. Project team C. Project manager D. Customers/end/users

B. Project team Agile teams are self managing, and thus make all planning decisions.

Quality touches every part of the project and is not just limited to the Quality Management area. There are two outputs of Quality that are used by other Knowledge Areas such as Validate Scope and Close Project or Phase. Those two outputs are: A. Verified deliverables; quality metrics B. Quality reports; verified deliverables C. Quality control measurements; quality reports D. Quality metrics, quality reports

B. Quality reports; verified deliverables Quality reports are used in closing projects as well as while monitoring and controlling project work. And both quality reports and verified deliverables are used as input to Validate Scope. The other choices are viable, but they are not called out specifically by PMBOK Guide.

You are managing a project which is wrapping up. One of your team members, Luke, comes to you to voice some frustrations about his role in the project. He feels that his job on the project was never clearly established, and that he was never provided with any concrete goals. Even though the team had been working together for a few months he never felt trusted to try his strengths and truly excel during the project. If this is true, you may have done a poor job with which of the following? A. Team Performance Assessments B. Roles and Responsibilities C. Team-Building Activities D. Communication Methods

B. Roles and Responsibilities Roles and responsibilities are a component of the resource management plan. The question suggests poor planning was done for project resources, this can lead to confusion is roles and doubts in expectations. PMBOK Guide Sixth Edition (2017) PMI/PMI/9.1.3.1 Resource Management Plan/Roles and Responsibilities/pp. 318-319

A customer indicates that they would like to add functionality. Although this request is made during the execution stage of this waterfall project, the project manager finds that the cost for the addition would be small. The additional functionality, if successful, would increase the opportunity for profit. What should the project manager do next? A. Agree that the functionality would offer a great opportunity, but the proposed addition is out of scope. B. Schedule a meeting with the company's management team, and present the customer's request for additional functionality as a good opportunity. C. Move forward, using the project's management reserve to add the requested functionality. D. Add it to the product backlog and prioritize it against the other components of the project.

B. Schedule a meeting with the company's management team, and present the customer's request for additional functionality as a good opportunity. By building a small increment and then testing and reviewing it, the team can add value in terms of new functionalities or additional awarded projects. An important factor for a successful implementation and value capture is the response time of the team.

On your project, a deliverable was recently created but had to be reworked due to a defect. You traced the root cause to an employee who was operating off an old version of the project management plan. As a project manager, how should you proceed? A. Fire the employee for not following protocol B. Show the employee where to find the most up-to-date copy of the plan and verify they are receiving updates C. Verify the employee is receiving notifications from the project management information system (PMIS) of any changed plans according to the change control board (CCB) D. Conduct training with all team members

B. Show the employee where to find the most up-to-date copy of the plan and verify they are receiving updates As the project manager, you want to ensure the employee is receiving updates and they know how and where to retrieve them from. Terminating the employee is an extreme measure that will add risk to your project. The change control board (CCB) does not dictate version control, they approve or reject changes. Conducting training for the entire team is a vague statement that may not identify the root cause of the problem.

When negotiating contracts with customers involving agile projects, a collaborative approach is recommended, since it emphasizes win-win between the buyer and seller vs. win-lose arrangements. Which of the following is not a characteristic of a collaborative approach? A. Emphasizing value delivered B. Single document contract C. Graduated time and materials D. Fixed price increments

B. Single document contract A multi-tiered contract structure provides for a master agreement to cover fixed items such as warranties and remedies, and including other elements that are more subject to change in other documents (e.g., schedule of services).

You are planning your schedule for a large project and you know that you want to hold regularly scheduled status meetings to keep the sponsor and key stakeholders apprised of the project progress. All of the following are true regarding status meetings except for which one? A. Status meetings are an example of interactive communication. B. Status meetings should be held every day to ensure nothing is missed. C. Status meetings are a way to formally exchange information and update the stakeholders on the project status. D. Status meetings should be held throughout the project at regularly scheduled intervals.

B. Status meetings should be held every day to ensure nothing is missed. Of the answer options listed, B is the best. Status meetings are considered interactive communication as they involve two or more parties performing multi-directional exchange of information.

A deliverable does not meet the agreed-upon quality specifications and is rejected by the customer. After investigation, the project team finds that there is a fault with the vendor-provided parts, and this fault caused the issue. The vendor says that they cannot correct the situation. What is best to review before deciding on the next steps? A. What alternative vendors are available B. The contract agreements and procurement management plan C. The internal quality assurance reports D. The resource management plan

B. The contract agreements and procurement management plan A contract is a mutually binding agreement that obligates the seller to provide the specified products, services, or results; obligates the buyer to compensate the seller; and represents a legal relationship that is subject to remedy in the courts.

While communicating an issue with the project with one of the stakeholders, they begin crossing their arms and becoming very defensive with you. You require their support on this project and they are known to be passive aggressive. How should you proceed? A. They concur with your thoughts on the issue and they are frustrated with the engineers for causing this problem B. They think you are blaming them for the issue and you should adjust your approach immediately and focus on how much they have done for the project C. Document the stakeholders response on the lessons learned register D. Change the topic of the conversation and try again tomorrow

B. They think you are blaming them for the issue and you should adjust your approach immediately and focus on how much they have done for the project Their body language changed during the conversation, this implies something stated began to bother the stakeholder. By focusing on the good things they have done it will make it easier to listen to how they can improve. Documenting it on the lessons learned register will not solve the issue currently at hand. Changing the topic will not solve the current issue and the issue still was not addressed.

Your company is new to agile concepts, and all people involved with projects have recently completed agile training. You have just been assigned as a project manager for your first agile project to develop a web front end for internal employees to register for health insurance and other benefits for the upcoming calendar year, with an expected delivery of 3 weeks. The project life cycle methodologies that are currently available are all based on a predictive/waterfall approach. What is the best course of action? A. Ask the project management office to develop a new agile methodology that will work for the new project B. Work with the project management office to develop a new or streamline an existing methodology that will work for the project C. Bring in outside agile consultants to help manage the project D. Create the initial work assignments for your resources since you are on a tight timeline, then work with the group to find the best way to get to agile as you move forward.

B. Work with the project management office to develop a new or streamline an existing methodology that will work for the project This is the hallmark of servant leadership.

Your company has won a contract to oversee the expansion of a major highway in your area. The project had been discussed for several years, and due to constant changes in the political climate, the controversial project was hard to get off the ground. The project has drawn criticism from nearby residential landowners, environmental groups, and taxpayers who deem it too expensive. Moreover, some environmental studies suggest that the subsurface in some areas may be inadequate to sustain the road, which may result in expensive detours being built around those spots. You are working on your risk management plan. What is the best way to proceed? A. Set aside reserves to cover the costs of these risks and others that have been identified B. Conduct a feasibility study to determine whether the project should go forward under these circumstances C. Document each category of risk in the Risk Management Plan to help ensure all risks are properly identified D. Ask the sponsor to assign an independent project team to assess and handle project risk

C. Document each category of risk in the Risk Management Plan to help ensure all risks are properly identified The Risk Management Plan should contain risk categories, which will help you systematically identify risks as you move through risk planning. A risk breakdown structure may be used to show risk categories, and this would be part of the Risk Management Plan. Remember, since you are working on the Risk Management Plan, you have not yet documented specific risks, as suggested in choice A.

A product passes 96% percent of the functional requirements defined in the scope. A customer is unwilling to accept the product since it did not pass 100%. A project manager is assessing what could have been done differently to avoid this. What should the project manager have done? A. Assessed and evaluated the test management plan B. Created the work breakdown structure (WBS) C. Ensured that the quality metrics were defined D. Reviewed the requirements documentation

C. Ensured that the quality metrics were defined Quality metrics are a description of a project or product attribute and how to measure it.

A client brings a project manager into a project during the execution phase. The project manager review the project scope for cost estimating purposes and notices some deliverables were missed in the work breakdown structure (WBS). Which step should the manager take next? A. Verify with the stakeholders to see if these missed deliverables should be added. B. Ask that the project management office (PMO) approve additional funds for this work. C. Include the work in the WBS, per the 100% rule, and recalculate the total project cost. D. Determine whether these deliverables are necessary by verifying with experts in the field.

C. Include the work in the WBS, per the 100% rule, and recalculate the total project cost. The WBS is a hierarchical decomposition of the total scope of work to be carried out by the project team to accomplish the project objectives and create the required deliverables. The WBS organizes and defines the total scope of the project and represents the work specified in the current approved project scope statement. This must always reflect the complete and accurate scope of work.

A customer is not satisfied with the delivered product, saying that it was not what they expected. The project manager is surprised, because the agile development team delivered the product several iterations early. What is one way that the project manager could have avoided this result? A. Required team sign off for each of the constant changes to the requirements B. Set aside more resources for product testing after changes are made C. Made sure that the customer was aware of the value of demos D. Ensured that the team fully participated in developing the project scope

C. Made sure that the customer was aware of the value of demos The customer was not engaged enough in the project. The iteration review is the best mechanism for the team to demonstrate working software and for the customer to give feedback to ensure that their expectations are met. PMBOK Guide Sixth Edition (2017) PMI/PMI/13.3 Manage Stakeholder Engagement/p. 524.

You are managing a large software project for a bank and are about 50% done with the work when a new stakeholder is identified. It turns out the stakeholder will be highly impacted by the new software project. You have a tight deadline and the requirements are already firmly set. What should you do? A. Ignore the stakeholder B. Meet with the stakeholder to inform her of your project C. Meet with the stakeholder to gather her requirements D. Find someone to blame for not identifying the stakeholder earlier

C. Meet with the stakeholder to gather her requirements The correct course of action would be to meet with the stakeholder and document her requirements. If time permits and the change is approved, you may still be able to incorporate her requirements. At the very least, you have engaged the stakeholder and attempted to satisfy her needs. Simply informing the stakeholder of your project and not gathering her requirements, as suggested in choice B, would not be appropriate either.

Which of the following should be done during the Develop Project Charter process? A. Identify risks and their risk responses to ensure the project plan is as stable as possible B. Decompose the scope of the project into more precise deliverables and activities to allow for the most accurate and confident estimate C. Perform a benefit analysis to validate project alignment with the organization's strategy and ensure the expected business value is doable D. Ensure all stakeholders have been identified with the appropriate strategies defined for engaging them effectively

C. Perform a benefit analysis to validate project alignment with the organization's strategy and ensure the expected business value is doable A is done in the Identify Risks and Plan Risk Responses processes, although some risks could have been identified during the develop project charter process. B is the Create work breakdown structure (WBS) and Define Activities processes. D is the Identify Stakeholders and Plan Stakeholder Engagement processes.

Your sponsor is currently evaluating multiple projects to determine which one would be the best to choose based on limited project resources. Project 1 is a five-year project with a net present value (NPV) of $115,000. Project 2 is a three-year project with an NPV of $85,000. Project 3 is a one-year project with an NPV of $74,000. Project 4 is a two-year project with an NPV of $55,000. Which project should be selected? A. Project 4 B. Project 2 C. Project 1 D. There is not enough information to make a determination

C. Project 1 When looking an NPV, the years have already been taken into consideration. Projects should be judged on the highest NPV, regardless of the number of years involved. Project 1 has the highest NPV value.

Which of the following would be most helpful in determining the total duration of your project? A. Schedule Management Plan B. Work Breakdown Structure (WBS) C. Project Schedule Network Diagram D. Scope Statement

C. Project Schedule Network Diagram The project schedule network diagram displays the relationships among activities, and would therefore be the most helpful in determining the total duration of the project. The schedule management plan would tell you how to go about schedule planning, but it would not contain information about project duration. The work breakdown structure (WBS) and scope statement would contain information about project scope, but lack sufficient detail about individual activities to determine the project duration.

Who is responsible for approving the project management plan? A. Project manager and stakeholders B. Project manager and sponsor C. Project manager, sponsor and key stakeholders D. Sponsor only

C. Project manager, sponsor and key stakeholders The project management plan is the master plan from which a predictive project will be executed. It is the responsibility of all key parties (project manager, sponsor, and key stakeholders) to approve the project management plan.

Which of the following project life cycle methods is most suitable for delivering business value in the shortest timeframe? A. Six Sigma B. Waterfall C. Scrum D. Capability Maturity Model Integration (CMMI)

C. Scrum Scrum is one of several agile methods (The Agile Practice Guide (2017) PMI/PMI/2.2 The Agile Manifesto and Mindset/Figure 2-4. Agile is a Blanket Term for Many Approaches/p. 11). Agile delivers value in short increments, typically 2 weeks to 1 month in duration. All other methods typically take more time. Waterfall is a predictive approach. Six Sigma focuses on process improvement. Capability Maturity Model Integration (CMMI) focuses on maturity assessment.

When an organization is considering which project life cycle methodology is appropriate to use for a given project, a suitability filter spider web diagram or model is useful. Which of the following factors is not considered when using this type of model? A. Trust B. Team Size C. Sponsorship D. Criticality

C. Sponsorship The three broad factors include culture, project and team. Trust is one of the factors under culture, criticality is under project, and team size is under team. Sponsorship is important for supporting organizational change, but it is not part of the model.

Jackie is a project manager working with a team of more than 30 people. One of her subordinates tells her that the team members at the lowest level are lazy, and that they need to be supervised more closely. Jackie responds that the workers are as committed to the project as she is, that they are ambitious and self-disciplined and do not need to be watched every minute of the day. Instead, Jackie proposes that each worker should be given more responsibility. Which type of manager is Jackie? A. Maslow's hierarchy of needs theory B. Theory Z manager C. Theory Y manager D. Theory X manager

C. Theory Y manager According to McGregor's Theory of X and Y (but not Z), Jackie is a Theory Y manager. She trusts her employees to be self motivated and believes that they do not need to be watched every minute. Theory X managers feel the way the subordinate does in this questions -- they feel that workers are lazy and need to be watched constantly. Theory Z is a management theory where creating the expectation of having the job for life motivates them to perform their best. Maslow's hierarchy of needs theory says that individuals must meet the most basic needs before they can reach peak performance.

A project manager is creating an agenda and presentation materials for a kick-off meeting that will involve stakeholders across multiple geographic regions who will be working on a global project. Which of the following would be the MOST important step for the project manager to ensure that the stakeholders are engaged? A. Create slides for the kick-off meeting to address stakeholder involvement. B. Survey stakeholders' preferred meeting dates and times to accommodate the teams in different time zones. C. Verify that all required participants or their delegates will be available to attend. D. Review the kick-off session agenda and materials for overall clarity and concision.

C. Verify that all required participants or their delegates will be available to attend. None of the other options are viable without ensuring that all stakeholders are attending the project kick off meeting.

What are the primary benefit of the sprint review meeting? 1 - The analysis of what the team did well, and could be improved 2 - Elicit feedback on the work that has been completed 3 - Updating the sprint backlog 4 - The evaluation and delivery of project value 5 - Meeting daily to discuss the team's impediments A. 1 and 3 B. 2 and 3 C. 4 and 5 D. 2 and 4

D. 2 and 4 Option D is correct. The sprint review meeting focuses on demonstrating and delivering working software from the prior sprint that meets the acceptance criteria defined by the product owner.

Although a previously identified technical issue has been resolved, the project manager is warned that the same issue is likely to occur on similar projects. Which of the following should be part of the first step in responding to the warning? 1 - Update the issue log 2 - Inform the PMO 3 - Create a risk report 4 - A communication with the project sponsor 5 - Review the lessons learned register A. 3 and 5 B. 2 and 3 C. 1 and 4 D. 2 and 5

D. 2 and 5 The PMO is responsible for sharing information between concurrent projects and the lessons learned register is archived in the lessons learned repository at the end of each project. This can be reviewed by project managers in future projects to avoid past project issues. Updating the issue logs for future project cannot be done, as these projects are in the future.

Which of the following would not typically be included in a team charter for an agile project? 1 - Group norms for meeting times, durations and formats 2 - Core and flexible work hours 3 - Project vision 4 - Working agreements 5 - Corporate mission 6 - Team member behavior expectations A. 1, 2, and 4 B. 2 and 4 C. 3, 5, and 6 D. 3 and 5

D. 3 and 5 The project vision is typically included in the project charter - not the social contract for the agile team (team charter).

During a retrospective, your team determines the most of their impediments stem from a single stakeholder who is abusing their authority. The project lead notes this particular stakeholder has never been fond of this project and has been very vocal about it being a bad decision on the sponsor. He also states this will lead the company in the wrong strategic direction. How could this have been avoided? A. Conducted a stakeholder analysis and created a stakeholder engagement assessment matrix B. Created a salience model and inform the stakeholder that we are here to do a job regardless of our personal feelings C. Informed the sponsor at the first instance and have them get buy-in from the stakeholder D. After conducting an analysis, determined their engagement and build a relationship with the stakeholder to get their buy-in

D. After conducting an analysis, determined their engagement and build a relationship with the stakeholder to get their buy-in By assessing their power, interest, and impact the project manager could then determine the appropriate engagement level required by the stakeholder. Then they must effectively engage them according to the stakeholder engagement plan and communication management plan.

You have just finished a release cycle on your agile project and things are going GREAT! The new features have been an instant success and the marketplace is waiting with high anticipation to see what you will bring them next. Which of the following is not true regarding the sprint planning meeting? A. The sprint goal, which includes features and user stories in the sprint backlog, is created B. Backlog items are prioritized, usually by the highest value C. It is typically attended by the product owner and agile team D. All tasks from decomposed user stories are created

D. All tasks from decomposed user stories are created Option D is correct. Usually, only the highest priority stories are decomposed into tasks during the sprint planning meeting. These will be the initial tasks performed during the upcoming sprint. The remaining stories will be decomposed later during the sprint. Answers A, B, and C are all true regarding the sprint planning meeting.

You are an agile coach in an organization that is starting to implement agile. What can be done to ease the transition to agile while beginning to satisfy certain aspects of the organization's current project management governance model? A. Use agile friendly reports and metrics to report project progress B. Educate your project management office on the benefits of agile C. You must change the current project governance model before implementing agile D. Alter your project reporting to fit governance expectations

D. Alter your project reporting to fit governance expectations Option D is correct. As an agile coach, you could alter your agile friendly information (burn up or burn down charts) to the required format (for example, a Gantt chart). Additionally, you could use story points, burn rate, and actual costs to provide earned value metrics.

You are running an agile project and many of the people on the team are relatively new to agile concepts. You are concerned about how well the team will adapt and perform over the long run, and want to help the team improve as it delivers each iteration. What is the best practice for doing this? A. Facilitate daily stand-up meetings B. Coach and mentor each team member throughout the project C. Have an independent party conduct an audit of the agile process and its effectiveness

D. Conduct retrospectives at the end of each iteration The retrospective is the most important practice for the team to learn about, improve, and adapt to the process. You might still do the other things, but the retrospective is the most important one for long term gains.

Which of the following is not true regarding the team charter? A. It sets the ground rules for team behavior B. It establishes values such as sustainable pace and core hours C. It defines working agreements including the definition of "ready" and "done" D. It is not considered a social contract

D. It is not considered a social contract Option D is correct. Answers A, B, and C are true regarding the team charter. However, the team charter is considered a social contract between the project team members.

You have identified several risks on your project but there is one particular risk that would result in a $100,000 loss should it materialize. Management has suggested a couple of solutions for you to explore that would allow you to minimize the impact should this risk occur, however, you are adamant that an insurance policy is the best way to proceed. Which of the following is true? A. Management is trying to mitigate the risk whereas you are trying to transfer the risk. Mitigating would be the better option since you can share the financial burden with the other partner organization. B. Management is trying to transfer the risk whereas you are trying to mitigate the risk. Mitigating would be the better option since you can share the financial burden with the other partner organizations. C. Management is trying to transfer the risk whereas you are trying to mitigate the risk. Transferring the risk would be the better option in this case since it involves the total financial loss should the risk materialize. D. Management is trying to mitigate the risk whereas you are trying to transfer the risk. Transferring the risk would be the better option in this case since it shifts the entire financial loss should the risk materialize.

D. Management is trying to mitigate the risk whereas you are trying to transfer the risk. Transferring the risk would be the better option in this case since it shifts the entire financial loss should the risk materialize. Management's approach of minimizing the impact of the risk with another solution is Mitigation whereas your approach of taking out an insurance policy is Transfer (because the insurance company would bear the risk of the loss). Taking out an insurance policy to protect from the large financial loss would be the best approach, because the amount is so significant.

Approaches to change will vary with the project lifecycle you are using. In a predictive project the project manager's attention is BEST spent doing which of the following? A. Informing the sponsor and stakeholders of changes B. Tracking and recording changes C. Building relationships with the change control board members D. Preventing unnecessary changes

D. Preventing unnecessary changes As a project manager you should try to be proactive and prevent changes as much as possible

Which of the following statements is true regarding risk management? A. Risks are events that have occurred on a project, and may be helpful (opportunities) or hurtful (threats) B. Risks might be opportunities to the project objectives, but only risks that are threats to the project objectives need to be considered C. Risks that are perceived more as rewards to the organization than negative consequences should be accepted D. Project risks are uncertain events

D. Project risks are uncertain events Risks are always uncertain events and need to be considered whether they are positive or negative. Even risks that involve perceived rewards should be analyzed.

You have been hired as a new project manager for the new football team for your state. You have taken over a project that is 50% complete, but you notice the work being done does not match what is written in the scope baseline. The sponsor informs you that many changes have occurred and to just roll with it. What most likely is the cause of this? A. The last project manager failed to do change control and this is why they were fired B. The change log has not been updated properly C. The organization lacks a configuration management plan D. Scope creep has most likely happened due to the lack of formal change control procedures

D. Scope creep has most likely happened due to the lack of formal change control procedures Formalized change control procedures reduce the chance of scope creep. A good process identifies how changes will be documented, implemented, approved, and validated. They also will conduct configuration management to ensure updates to the versions of the documents are available to all stakeholders.

There are three deliverables in the project manager's IT transformation project: Deliverable 2 has strictly scheduled milestones, with no expected variation in the timeline. The project manager expects that Deliverable 3 will see quickly changing requirements during development. Which lifecycle model should be used to meet the project's requirements? A. Select a fully agile project model, with a common user story and three-week sprints. B. Select a waterfall project model for deliverables, with firm milestones and change control procedures. C. Select a hybrid project model, where Deliverable 2 is positioned as a single agile sprint embedded in an overall waterfall project. D. Select a hybrid project model, where Deliverable 2 is positioned as a single waterfall phase embedded in an overall agile project.

D. Select a hybrid project model, where Deliverable 2 is positioned as a single waterfall phase embedded in an overall agile project. In this project, two deliverables have unknown durations or changing requirement sets appropriate for agile. The migration deliverable 2 requires tight schedule control and external dependencies appropriate for a waterfall project.

You know a small portion of your project work will have to be performed by contract workers from an external organization, however, you cannot currently create a precise statement of work for this procurement. Which type of contract would be best suited for this situation? A. Cost Plus Incentive Fee (CPIF) B. Fixed Price (FP) C. Cost Plus Percentage of Costs (CPPC) D. Time and Materials (T&M)

D. Time and Materials (T&M) Time and materials (T&M) contracts are best used in cases where precise statements of work cannot be readily prescribed. They are most commonly used for staff augmentation, as indicated in the facts of this question. You should consider using a not-to-exceed clause to help prevent runaway costs when using this contract type.

You are the project manager for a large pharmaceutical company that recently partnered with another organization that reduces packaging costs for your organization under the auspice of a cost-reimbursable contract. After a risk workshop meeting, you find out two team members do not agree on a proposed solution for a risk to cost. You decide to let them come to a resolution on their own, but you document it on the issue log. What conflict management technique did you implement? A. Direct/Force B. Retreat/Avoid C. Compromise/Reconcile D. Withdraw/Avoid

D. Withdraw/Avoid Withdraw/Avoid is retreating from the conflict. The project manager is allowing the team members to resolve the issue themselves. By putting it on the issue log they are ensuring they will follow up on the conflict to make sure it is resolved.


Related study sets

Épocas y Movimientos Literarios

View Set

Romeo and Juliet Characters Act 1-2

View Set

Additive Inverse and Identity, Multiplicative Inverse, Identity, and property of zero Commutative Associative Distributive Properties

View Set

Nutrition Ch. 6: Protein and Amino Acids

View Set

ARRT Prep questions: Extremities

View Set

Chapter 25 - Fluid and Electrolytes

View Set